The formula in cell C5 is "=SUMPRODUCT($A$1:$C$1,A2:C2)". If one copies and pastes the formula from the cell C5 into the cell C6, what numerical value will appear in the cell C6?

Answers

Answer 1

Answer:

See Explanation

Step-by-step explanation:

Given

Formula: =SUMPRODUCT($A$1:$C$1,A2:C2)

Required

Determine the numerical value that will appear

The question seem incomplete; however, I'll give a general explanation

FIrst, when the formula is copied from C5 to C6, A2 and C2 changes to A3 and C3 respectively.

This is because they are relative references and they behave on a relative position.

This implies that, when the formula is in cell C5, the formula considers cells A2 and C2 when executing the formula;

However, C6 will consider cells A3 and C3, respectively.


Related Questions

10.
J
A
B
0
L
с
K
Not drawn to scale
JK, KL, and are all tangent to circle O. JA = 9, AL = 10, and CK = 14. What is the perimeter of AJKL?
66 units
46 units
O 33 units
38 units

Answers

Answer:

66

Step-by-step explanation:

The figure of triangle JKL is attached. JA = 9, AL = 10, and CK = 14.

According to two tangent theorem, the tangent to a circle that meets at the same point is equal to each other. Therefore:

AJ = BJ, AL = CL, CK = BK.

Since AJ = 9, BJ = AJ = 9

AL = 10, CL = AL = 10

CK = 14, BK = CK = 14.

Therefore the lengths of the triangle sides are:

JL = AJ + AL = 9 + 10 = 19

JK = BJ + BK = 9 + 14 = 23

KL = CL + CK = 14 + 10 = 24

The perimeter of the triangle is the sum of all its sides, it is given as:

Perimeter = JL + JK + KL = 19 + 23 + 24 = 66

During a catered lunch =, an average of 4 cups of tea are poured per minute. The lunch will last 2 hours. How many gallons of tea should the caterer bring if there are 16 cups in one gallon?

Answers

Answer:

30 gallons of tea

Step-by-step explanation:

We are looking at the average of cups of tea per minute but we are given the time frame of lunch in hours, so first, we have to convert the hours to minutes:

There are 60 minutes in 1 hour and lunch is 2 hours long.  So, multiply 60 by 2 to get 120 minutes total.

Next, we have to find out the number of cups of tea poured during the lunch.  We have been told already that an average of 4 cups of tea are poured a minute.

Therefore, multiply 4 by the total number of minutes for lunch.  You will multiply 4 by 20 to get 480 cups of tea poured in total during the catered lunch.

Finally, we have to see how many gallons of tea the caterer should bring.  We should know that there are 16 cups in one gallon.

That means we have to divide the total number of cups poured by 16.  Divide 480 by 16 to get 30 gallons of tea that the caterer should bring.

20 POINTS!!! Use the quadratic formula above to solve for h(t) = -4.9t^2 + 8t + 1 where h is the height of the ball in meters and t is time in seconds. Round to the nearest hundredth second!

Answers

Answer:

Two solutions: -0.12 and 1.75.

Step-by-step explanation:

The quadratic formula is:

[tex]\begin{array}{*{20}c} {\frac{{ - b \pm \sqrt {b^2 - 4ac} }}{{2a}}} \end{array}[/tex]. Assuming that the x² term is a, the x term is b, and the constant is c, we can plug the values into the equation.

[tex]\begin{array}{*{20}c}{\frac{{ - 8 \pm \sqrt {8^2 - 4\cdot-4.9\cdot1} }}{{2\cdot-4.9}}} \end{array}[/tex]

[tex]\begin{array}{*{20}c}{\frac{{ - 8 \pm \sqrt {64 + 19.6} }}{{-9.8}}} \end{array}[/tex]

[tex]\begin{array}{*{20}c}{\frac{{ - 8 \pm \sqrt {83.6} }}{{-9.8}}} \end{array}[/tex]

[tex]\begin{array}{*{20}c}{\frac{{ - 8 \pm \sqrt {9.14} }}{{-9.8}}} \end{array}[/tex]

[tex]\frac{-8 + 9.14}{-9.8} = -0.12[/tex]

[tex]\frac{-8-9.14}{-9.8} =1.75[/tex]

Hope this helped!

Emma changed £500 into rand before going on holiday to South Africa.
The rate of exchange at the time was £1 = 10.4 rand.
Emma spent 4000 rand on holiday. When she got home, she changed her leftover rand into pounds.
The exchange rate was now £1 = 9.8 rand. How much money did she get back in pounds?

Answers

Answer:

I'm sorry but I can give exact numbers but I would like to help work it out so...

Step-by-step explanation:

So overall she had £500 to start with

And £1 is equal to 10.4 rand

So you would divide 100 by 10.4 and get the potential difference between the average of money which she has then because she spent 400 rand in holiday you would divide 400 by the amount of the potential difference which was given then change that back to pounds

Hope this helps

If this seems incorrect please comment and I will change my answer thanks:)

What are the solutions to the quadratic equation below?
4X2 +28x + 49 = 5

Answers

4* 2 + 28x + 49 = 5
8 + 28x + 49 = 5
28 + 57 = 5
- 57 -57
———————————
28x = - 52
———————-
28x = 28

x = - 13/7

1) Multiple the numbers
2) Add the numbers
3) Add the same terms to both sides of the equation
4) Simplify
5) Divide both sides of the equation
6) Simplify

hey can someone help me out here because i dont know none of this

Answers

Answer:

x = 0

Step-by-step explanation:

In order for this to be a function there has to be an x and y pair.  You cant have 2 different x values correlate to 1 y value.  The only number not used on the table of value is 0

Therefore the value of x is 0

Answer: B. x = 0

==================================================

Explanation:

The table shows the inputs of -5, -1, x, 1 and 3. Ignore the x for now. The numerical inputs are -5, -1, 1, 3

If we have any input repeat itself with a different paired y value, then we will not have a function.

So if x = -5, then we don't have a function. This is because x = -5 is already paired with y = 4 in the top row. We can't have the input x = -5 lead to y = 0 at the same time. Any input must lead to exactly one output only. So this rules out choice A as a possible answer.

Choice C and choice D are eliminated for similar reasons as well. This leaves choice B. We don't have x = 0 yet, so it is a valid possible input. We can pick any thing we want for x as long as its not already done so in the table.

Numbers 1-6 please and thank you This is really hard and I really really need help. I appreciate all the help I can get.

Answers

Area of prism = base area × altitude

1. (2x²-10)(x+4)

= 3x³-2x - 40

2. Base area = 2πr

Volume = (2πr)(r²+ 5r)

=2πr³ + 10πr²

3. Base area=½(6)(x-4)(x+3)=3(x-4)(x+3)

Volume= 3(x-4)(x+3)(⅓)

=(x-4)(x+3)= x² - x -12

4. Base circumference= 10π

Base radius = 10π/(2π) = 5

Base area = πr² = 25π

Volume = 25π(3x²-2x)

=125πx²-50πx

5. Volume = 3π√50

=15π√2

6. Base diameter = 16

Base radius = 16/2 = 8

Base area = 2πr = 16π

Volume = 16π(23a²)

=368πa²

4+2p=10 (3/4p-2) solve for p

Answers

Answer:

p = 48/11 or 4.36

Step-by-step explanation:

4 + 2p = 10(3/4p - 2)

distribute the 10 on the right side of the equation

4 + 2p = (15/2p - 20)

multiply both sides by 2

8 + 4p = 15p - 40

move the terms

48 = 11p

p = 48/11

(sorry if this question is already answered, brainly is glitching out for me)

Answer:

p=6

I got it right on Kahn Academy

Solve for x: 2x+1= -3x+36

Answers

Answer:

x = 7

Step-by-step explanation:

2x + 1 = -3x + 36

2x + 3x + 1 = -3x + 3x +36

5x + 1 = 36

5x + 1 - 1 = 36 - 1

5x = 35

5x/5 = 35/5

x = 7

Answer:

first you would add 3x to -3x and 2x, then you would get 5x+1=36. Then you subtract 1 from 1 and 36. Then you get 5x=35. Then you divide by 5 to get the answer 7. so your answer is x=7

Step-by-step explanation:

hope this helps

Marsha wants to buy carpet to cover her whole living room, except for the tiled floor. The tiled
floor is 4 ft by 2 ft. Find the area the carpet needs to cover. 1)
Solve on paper. Then check your work on Zearn. 1)
1
11
3
7
tiled floor

Answers

Answer:

68.06 sq. feets

Step-by-step explanation:

Area of room's floor

11.33 × 7 = 79.33

Area of tiled floor

4.83 × 2.33 = 11.278

Area of floor needed to be carpeted

79.33-11.278= 68.06

what is nine and forty-two hundredths

Answers

Answer:

9.42

Step-by-step explanation:

Breaking the phrase down:

'Nine' would be the number 9 in the ones place.

'And' represents the decimal in a number. ('.')

'Forty-Two Hundredths" is 0.42.

So, "nine and forty-two hundredths" would be 9.42.

Hope this helps.

please help on 30–31

Answers

Step-by-step explanation:

30-option c

because only crows r black in appearance

31-option d

thats the option which represents the question asked

One week, Daniel earned $554.30 at his job when he worked for 23 hours. If he is paid the same hourly wage, how many hours would he have to work the next week to earn $939.90?

Answers

Answer:

39 hours

Step-by-step explanation:

First, let's find the rate of money per hour:

$554.30 / 23 hour

= $24.1 / 1 hour

Daniel earns $24.10 every hour he works.

To find the hours that Daniel has to work to earn $939.90, divide it by 24.10:

$939.90 / 24.10

= 39

Now, we can check:

$24.10 * 39

= $939.90

Hope this helps!  Please tell me if I was incorrect!

He would work 39 hours next week.

Please Help! Three times the quantity of a number increased by 7 is equal to the same number decreased by 15

Answers

I guess it's x=-11
3x+7=x-15
3x-x=-15-7
2x=-22
X=-11

The equation is written as 3x + 7 = x - 15. And the solution of the equation will be negative 11.

What is the solution to the equation?

The allocation of weights to the important variables that produce the calculation's optimum is referred to as a direct consequence.

Multiple times the amount of a number expanded by 7 is equivalent to a similar number diminished by 15.

Let the number be 'x'.

The three times the number plus 7. Then the expression is given as,

⇒ 3x + 7

The number 'x' is decreased by 15. Then the expression is given as,

⇒ x - 15

Both expressions are equal to each other. Then we have

3x + 7 = x - 15

3x - x = - 15 - 7

2x = - 22

x = - 11

The equation is written as 3x + 7 = x - 15. And the solution of the equation will be negative 11.

More about the solution of the equation link is given below.

https://brainly.com/question/545403

#SPJ2

Euphrosynelight needs 2x-7 while her friend needs 5x+2 how much in total

Answers

Answer:

7x-5

Hope this helped; mark brainliest if it did! :)

Ms. Ayala had 152 pencils. She divided the numbers of pencils equally among 13 students. What is the greatest numberof pencils Ms.Ayala could have given each student

Answers

Answer:

11

Step-by-step explanation:

she divides the number of pencils equally

so the number of pencils per student is 152/13

which is 11.692...

but she can't give 0.69 of a pencil so she gives only the whole part 11

and she keeps 9 pencils

She keeps 9 pecainsle

The Tama, Japan, monorail carries 92,700 riders

each day. If the monorail usually carries

5,150 riders per hour, how many hours does

the monorail run each day?

Answers

Answer:

The number of hours monorail run each day is 18.

Step-by-step explanation:

The total number of riders the monorail carry each day is:

N = 92700.

The number of riders the monorail carry per hour is:

n = 5150.

Compute the number of hours the monorail run each day as follows:

[tex]\text{Number of hours the monorail run each day}=\frac{N}{n}[/tex]

                                                                     [tex]=\frac{92700}{5150}\\\\=18[/tex]

Thus, the number of hours monorail run each day is 18.

PLEASE HELP f(x)=x^2 and g(x)=(x-3)^2+2 Describe how the graph of g(x) relates to the graph of its parent function, f(x). (HINT: Think about how f(x) was shifted to get g(x))

Answers

Answer:

The graph f(x) was shifted 3 units to the right and shifted 2 units up to get the graph of g(x).

Step-by-step explanation:

From the original graph to the transformed one, we can see that the transformations (x - 3) and + 2 were added to the equation.

The (x - 3) means that the x-value of the vertex will increase by 3, meaning that the graph will shift 3 units to the right.

The +2 will increase the y-value of the vertex by 2, meaning that the graph will move up 2 units.

So, the graph of g(x) relates to f(x) as it is a transformation 3 units to the right and 2 units upwards.

How many times does 5 go into 1,200??

Answers

Answer:

240 times.

Explanation:

24 times because,

If you divide 5 with 12k that's,

= 1200/5

= 240

Hence proved, 240 times.

You pull one card at random from a standard deck and you shuffle the remaining cards. Then you pull another card. Is the event independent or dependent?

Answers

Answer:

If an event is affected by previous events then it is a dependent event, while if an event is not affected by the previous event then it is an independent event.

Since we have replaced the card that we first drew from the deck, it wont affect the event of pulling a card second time.

So, we can say that it is an example of independent event.

What is the inverse of the function g(x)=-3(x+6)? g^-1(x)=

Answers

Answer:

[tex]g^{-1}(x)=-6-\frac{x}{3} =-\frac{x}{3} -6[/tex]

Step-by-step explanation:

First assign a letter "y" to g(x) and get rid of parenthesis on the right:

[tex]g(x)=-3\,(x+6)\\y=-3x-18[/tex]

Now, solve for "x":

[tex]3x=-18-y\\x=\frac{-18-y}{3}\\x=-6-\frac{y}{3}[/tex]

now replace y with x, and call x : [tex]g^{-1}(x)[/tex]

[tex]x=-6-\frac{y}{3} \\g^{-1}(x)=-6-\frac{x}{3}[/tex]

Answer:

-6 - [tex]\frac{x}{3}[/tex]

Step-by-step explanation:

Factor 75 - 95. a. 5(15 - 19) b. 5(19 - 15) c. 25(3 - 4) d. 25(4 - 3)

Answers

Answer:

a. 5(15-19)

Step-by-step explanation:

to factor out this expression you need to find the greatest common factor (GCF) in order to fully factor out the expression

the GCF of the number 75 and -95 is 5

divide both numbers by 5 to get 15 and -19

to finish out with the fully factored expression put 15-19 inside parenthesis and put a 5 outside of the parenthesis as shown below:

5(15-19)

Answer:

a.  5(15  -19)

Step-by-step explanation:

15*5 = 75

-19*5 = -95

Factor is:

5(15 -19)

A golf ball is hit off a tee toward the green. The height of the ball is modeled by the function h(t) = −16t2 + 96t, where t equals the time in seconds and h(t) represents the height of the ball at time t seconds. What is the axis of symmetry, and what does it represent? t = 3; It takes the ball 3 seconds to reach the maximum height and 6 seconds to fall back to the ground. t = 3; It takes the ball 3 seconds to reach the maximum height and 3 seconds to fall back to the ground. t = 6; It takes the ball 6 seconds to reach the maximum height and 3 seconds to fall back to the ground. t = 6; It takes the ball 6 seconds to reach the maximum height and 6 seconds to fall back to the ground.

Answers

Answer:

t = 3; It takes the ball 3 seconds to reach the maximum height and 6 seconds to fall back to the ground.

Step-by-step explanation:

To find the axis of symmetry, we need to find the vertex by turning this equation into vertex form (this is y = a(x - c)² + d where (c, d) is the vertex). To do this, we can use the "completing the square" strategy.

h(t) = -16t² + 96t

= -16(t² - 6t)

= -16(t² - 6t + 9) - (-16) * 9

= -16(t - 3)² + 144

Therefore, we know that the vertex is (3, 144) so the axis of symmetry is t = 3. Since the coefficient of the squared term, -16, is negative, it means that the vertex is the maximum. We know that it takes the golf ball 3 seconds to reach the maximum height (since the t value of the vertex is 3) and because the vertex is on the axis of symmetry, it would take 3 more seconds for the ball to fall to the ground, therefore it takes 3 + 3 = 6 seconds to fall to the ground. The final answer is "t = 3; It takes the ball 3 seconds to reach the maximum height and 6 seconds to fall back to the ground.".

The time will be t = 3; It takes the ball 3 seconds to reach the maximum height and 6 seconds to fall back to the ground.

What is Function?

Function, in mathematics, an expression, rule, or law that defines a relationship between one variable (the independent variable) and another variable the dependent variable.

To find the axis of symmetry, we need to find the vertex by turning this equation into vertex form (this is y = a(x - c)² + d where (c, d) is the vertex). To do this, we can use the "completing the square" strategy.

h(t) = -16t² + 96t

= -16(t² - 6t)

= -16(t² - 6t + 9) - (-16) * 9

= -16(t - 3)² + 144

Therefore, we know that the vertex is (3, 144) so the axis of symmetry is t = 3. Since the coefficient of the squared term, -16, is negative, it means that the vertex is the maximum.

We know that it takes the golf ball 3 seconds to reach the maximum height (since the t value of the vertex is 3) and because the vertex is on the axis of symmetry, it would take 3 more seconds for the ball to fall to the ground, therefore it takes 3 + 3 = 6 seconds to fall to the ground.

The final answer is "t = 3; It takes the ball 3 seconds to reach the maximum height and 6 seconds to fall back to the ground.".

To know more about Function follow

https://brainly.com/question/25638609

#SPJ2

Given that ΔABC is a right triangle with a right angle at C, if tan A = [tex]\frac{5}{4}[/tex], find the value for tan B.

A. tanB = [tex]\frac{3}{4}[/tex]
B. tanB = [tex]-\frac{4}{5}[/tex]
C. tanB = [tex]\frac{4}{5}[/tex]
D. tanB = [tex]-\frac{5}{4}[/tex]

Answers

Answer:

C

Step-by-step explanation:

tan A = [tex]\frac{5}{4}[/tex] = [tex]\frac{opposite}{adjacent}[/tex] , thus

The opposite side is the adjacent side for B and the adjacent side is the opposite side for B, thus

tan B = [tex]\frac{4}{5}[/tex]

solution for 2x is equal to 10​

Answers

Answer:

The answer is 5

Step-by-step explanation:

divide 10 by two and get 5

Answer:

[tex]x = 5[/tex]

Step-by-step explanation:

We have the equation [tex]2x = 10[/tex], we can try and isolate x by dividing both sides by 2.

[tex]2x \div 2 = 10\div2\\x = 5[/tex]

Hope this helped!

Select the equivalent expression. (8^-5/2^-2)^-4 = ?
Choose 1 answer:
A. 1/8*2^2
B. 2^6/8^9
C. 8^20/2^8

Answers

Answer:

C. 8^20/2^8

Step-by-step explanation:

(8^5/2²)^4

8^20/2^8

Top Hat Soda has 300,000 milliliters of cola to bottle. Each bottle holds 500 milliliters. How many bottles will the cola fill?

Answers

Answer:600

Step-by-step explanation:

300,000/ 500 =600

Please answer ASAP!

Type your response in the box. Jack and Mia are playing a game with pick-up sticks. Mia places a pile of 100 pick-up sticks on the table. Forty of the sticks are black, and the rest are brown. She randomly splits all the sticks into two piles—one on Jack’s left and one on his right. Mia tells Jack that there are 44 brown pick-up sticks in the pile on his right. Jack looks at the pile of pick-up sticks on his left and estimates that it contains 44 sticks in all. Now Mia blind folds Jack and asks him to choose a stick at random. Jack knows that if he selects a black pick-up stick, Mia will treat him to dinner at his favorite restaurant. If he picks a brown one, then he will treat Mia to dinner at her favorite restaurant. Mia gives Jack three options for selecting:

Choose randomly from the pile on the left.

Choose randomly from the pile on the right.

Push the piles back together and choose randomly from the entire pile.

Which option should Jack choose so that Mia treats him to dinner at his favorite restaurant? Explain your answer.

Answers

Answer: Choose randomly from the pile on the left.

Step-by-step explanation: The ratio of brown to black sticks on the left pile is 16:28 and on the right pile is 44:12. Therefore, jack should choose from the left side because there is a higher chance in picking a black stick.

Arc length practice

Answers

Answer:

[tex]\large\boxed{s = 4\pi}[/tex]

Step-by-step explanation:

The arc length is determined by the formula [tex]s=r\theta[/tex], where s is the arc length, r is the radius, and [tex]\theta[/tex] is the value of the central angle (in radian formatting).

By substituting the values for the radius and the central angle, you can solve for the arc length.

[tex]\text{The radius is half of the diameter -} \: \boxed{\frac{4}{2}=2}[/tex].

The central angle is converted to radian form by multiplying the angle in degrees by the fraction of π/180 - 360° * π/180 = 360π/180 = 2π.

Now, substitute the values and solve for s.

s = (2)(2π)

[tex]\large\boxed{s = 4\pi}[/tex]

Sandy's older sister was given $2,400 and was told to keep the balance of the money after sharing with her siblings. Give Sandy exactly $350. Write Sandy's portion

Answers

Answer:

Sandy's portion = [tex]\mathsf{\dfrac{7}{48}}[/tex]

Step-by-step explanation:

Sandy's older sister was given $2,400

She was told to keep the balance of the money after sharing with her siblings.

She gave  Sandy exactly $350

The objective is to write Sandy's portion.

Sandy's portion will be the ratio of the amount given to Sandy divided by the total amount at her sister disposal.

Let Sandy's older sister be y,

So, y = 2400

Sandy's portion = [tex]\dfrac{350}{2400}[/tex]

Sandy's portion = [tex]\dfrac{35}{240}[/tex]

Sandy's portion = [tex]\mathsf{\dfrac{7}{48}}[/tex]

Other Questions
How does Paine support his idea that there is no country on the globe as capable of raising a eet as America? please tell me a story in simple word about the following title: "Pride hath a fall" Which equation represents the line that is perpendicular to y=3/4x+1 and passes through (-5,11)Will give brainliest!! I tried something similar to the notation of (x+2)^7, etc, did not get close at all, how would this be solved? discribe and explain the agency theory in your own words. you answer should also include the agency problem Please answer this question now An average skater averages 11 m/s over the first 5 seconds of a race. find the average speed required over next 10 seconds to average 12 m/s overall. Clinton is having a coin drive. Each class has a goal of 600 pennies.Layla has already collected 338 pennies. How many more pennies does Layla need to collect to reach their goal. PLEASE SHOW YOUR WORK I WILL MARK YOU BRAINLIEST PLEASE AND EXPLAIN HOW YOU GOT YOUR ANSWER Human BonesName three components of bones and describetheir function. Which sentence illustrates correct pronoun-antecedent agreement? A. Did most of the players sink their three-point shots? B. Neither Martin nor Cesar made most of their foul shots. C. Everyone who tried out for the basketball team had their hearts set on playing center. D. Each of the players must provide their own uniforms. On May 10, 2017, Vaughn Co. enters into a contract to deliver a product to Greig Inc. on June 15, 2017. Greig agrees to pay the full contract price of $1,990 on July 15, 2017. The cost of the goods is $1,310. Vaughn delivers the product to Greig on June 15, 2017, and receives payment on July 15, 2017. Prepare the journal entries for Vaughn related to this contract. Either party may terminate the contract without compensation until one of the parties performs. The first electric, general-purpose computer, ENIAC, was programmed by calculating algorithms on paper entering code directly into the computer flipping switches by hand using MS-DOS as the operating system what's the answer to this Determine the magnitude of the force that must be applied to a wooden block whose weight has a magnitude of 8N, so that it acquires an acceleration of 0.5 m / s why is carbon iv oxide used in fire extinguishers The two-way frequency table below shows data on years working with the company and college degree status for Tom's coworkers. Complete the following two-way table of row relative frequencies. (If necessary, round your answers to the nearest hundredth.) What is an empirical formula? Which parent functions have an intercept at (0,0)Choose all that are correct.LinearQuadraticRadicalAbsolute ValueRationalExponentialLogarithmic (Log)CubicCube Root Why would the basic nature of humans be a topic of discussion for these philosophers?(Thomas Hobbes,John Locke,Charles-Louis Montesquieu, and Jean-Jacques Rousseau)PLEASE HELP!! I need help with multistep equations